What are your chances of acceptance?

Calculate for all schools, your chance of acceptance.

Duke University

Your chancing factors

Extracurriculars.

thesis statement dbq example

How to Write the Document Based Question (DBQ)

Do you know how to improve your profile for college applications.

See how your profile ranks among thousands of other students using CollegeVine. Calculate your chances at your dream schools and learn what areas you need to improve right now — it only takes 3 minutes and it's 100% free.

Show me what areas I need to improve

What’s Covered:

What is the document based question, steps to writing an effective dbq, how do ap scores affect my college chances.

If you’re taking a history AP exam, you’ll likely encounter the Document Based Question (DBQ). This essay question constitutes a significant portion of your exam, so it’s important that you have a good grasp on how best to approach the DBQ. In this post, we’ll cover what exactly a document based question is, and how to answer it successfully.

A Document Based Question (DBQ) is a measure of the skills you learned in your AP classes in regard to recalling history and analyzing related documents. These documents can be primary or secondary sources, and your responses are expected to be in the form of an essay. Your ability to relate the context of documents to concepts beyond the given text and creating meaningful connections between all your sources will help demonstrate your skills as a knowledgeable writer.

The number of documents for a DBQ varies from exam to exam, but typically will fall between five to seven documents. The following AP exams will require you to write a DBQ:

AP U.S. History

AP European History

AP World History

We’ve listed the formats for each exam below, and keep in mind that the number of documents is prone to changing from year to year:

  • Up to seven Documents
  • One hour recommended time (includes 15-minute reading period)
  • Up to seven Documents 
  • 25% of total exam score

With that in mind, let’s jump right into how to craft a strong DBQ response!

We’ve summarized how to write an effective DBQ into the following five steps:

1. Read the prompt first

Though you may be tempted to jump into the documents right away, it’s very important that you first look at what exactly the prompt is asking for. This way, when you eventually look at the documents, your focus will be narrower. A DBQ tests your reading comprehension and analysis skills more than the content itself, making it very important to understand your prompt thoroughly.

2. Skim the document titles

Each document will contain vital information regarding the context, and it’s important to scout key words regarding dates, authors, and anything pertaining to the general sense of what the documents are about. Skimming through your documents like this could save time and allow you to form a more structurally sound thesis.

Let’s take a look at the following graph and figure out how to skim the figure:

thesis statement dbq example

This document was in a real exam from the AP World History free response questions in 2019. It’s important to pay attention to data provided and what context can be drawn from it. In this case, we’re provided with a graph that displays the life expectancy of a country in relation to the GDP per capita of said country. Being able to skim this graph and notice the common trends in the data points could provide convenient information into the context of the document, without any further intensive reading. 

For example, seeing how countries with a GDP below 4,000 to 5,000 have lower life expectancies already gives us a potential correlation between the two factors. We can use this information to start formulating a thesis, depending on what the prompt is specifically asking for.

Remember, just skim! Don’t worry about reading the entire document yet; this strategy can keep you calm and level-headed before tackling the rest of the document. Methods like this can make acing the AP World History DBQ less intimidating! 

3. Formulate a tentative thesis

A thesis is a statement that should be proved and discussed upon. It’s important to have a strong thesis as the foundation of your DBQ, as it guides the rest of your response in relation to the context. Understanding the difference between weak and strong theses will be imperative to your success, so here is an example of a weak thesis:

“The Cold War originated from some scenarios of conflict between Soviets and some groups of oppressors.” 

Such a thesis can be considered weak for its lack of specificity, focal point, and usability as a constructive tool to write further detail on the subject. This thesis does not take a clear stance or communicate to the reader what the essay will specifically focus on. Here’s how the same thesis can be restructured to be stronger and more useful:

“The Cold War originated from tense diplomatic conflicts relating to propaganda and conspiratorial warfare between the United States and the Soviet Union.”

The information that’s been included into the second thesis about the two groups involved with the Cold War gives you more room to build a structured essay response. In relation to the rubric/grading schema for this DBQ, forming a structurally sound thesis or claim is one of the seven attainable points. Being able to contextualize, analyze, and reason off of this thesis alone could provide for two to four points – this means that five out of seven of your points revolve around your thesis, so make sure that it’s strong! Doing all of this in your fifteen minute reading period is crucial as once this is set, writing your actual response will be much easier!

4. Actively read the documents

Simply reading a document doesn’t normally suffice for creating a well-written and comprehensive response. You should focus on implementing your active reading skills, as this will make a huge difference as to how efficient you are during your work process. 

Active reading refers to reading with an intention to grab key words and fragments of important information, usually gone about by highlighting and separating important phrases. Annotations, underlining, and circling are all great ways to filter out important information from irrelevant text in the documents. 

An example of where you might find important information via active reading is the description. Circle important names or dates to contextualize the document. If you still can’t find contextual value from the title, that’s totally fine! Just scope out the rest of the document in relevance to your thesis – that is, pinpoint the specific information or text that best supports your argument. Finding one or two solid points of interest from one document is usually enough to write about and expand upon within your essay. 

thesis statement dbq example

Discover your chances at hundreds of schools

Our free chancing engine takes into account your history, background, test scores, and extracurricular activities to show you your real chances of admission—and how to improve them.

5. Make an Outline 

If you like outlines, making one before writing your essay might prove helpful, just be aware of the time limit and act accordingly. 

Start with your introduction, then work on the rest of your essay. This way, you can make sure your thesis is clear and strong, and it will help the graders form a clear view on what the general consensus of your paper is. Make sure to include evidence with your thesis within each paragraph and cite only relevant information, otherwise your citations could come across as filler as opposed to useful content. Every commentary or point you make should be tied in some way to the documents.

Format each body paragraph and organize your essay in a way that makes sense to you! The graders aren’t really looking at the structure of your essay; rather, they want to see that you analyzed the documents in a way that is supportive of your essay. As long as you have content from the documents which prove your thesis, the order or manner in which you present them doesn’t matter too much. What’s more important is that your essay is clear and comprehensive. As you write practice DBQs, try having someone else read your essays to make sure that the format is easy to follow.

Keep all these key details in mind as you construct your own DBQ response, and you’re well on your way to writing an effective essay!

Your chances of admission are actually not really impacted by your AP scores; however, the AP classes you take are more important than the exam scores themselves, meaning the impact of your AP scores isn’t as big as you think . 

Instead, focusing on the AP classes on your transcript and the relevance of those classes to your future major is more impactful. For a further detailed understanding of the role AP classes play in regards to your college admissions, use CollegeVine’s free Admissions Calculator , which takes into account your GPA, standardized test scores, and more. 

Additional Information

To dive deeper into DBQs, AP classes, and learning how to tackle each exam check out other resources at CollegeVine:

  • Acing the Document Based Question on the AP US History Exam
  • Acing the AP World History Document Based Question
  • Ultimate Guide to the AP U.S. History Exam
  • Ultimate Guide to the AP European History Exam
  • Ultimate Guide to the AP World History Exam

Related CollegeVine Blog Posts

thesis statement dbq example

AP World DBQ Outline + Thesis Practice with Feedback

5 min read • june 18, 2024

Eric Beckman

Eric Beckman

Evan Liddle

Evan Liddle

Melissa Longnecker

Melissa Longnecker

DBQ Practice is very important when preparing for the AP World exam. It is recommended to write a short brief outline of your argument before writing your body paragraph.

Your task: In  20 minutes or less , read the documents and:

  • Outline arguments you would make, using LESS than a full sentence for each
  • List, but DO NOT describe, evidence, both documents and outside evidence
  • Write a thesis based on these arguments Note: on the actual exam spending more than 20 minutes on this will not leave you with enough time to complete what you plan

DBQ Revolutions in Americas Prompt

Use these documents to answer the following prompt:

Develop an argument that evaluates the extent to which revolutions in the Americas between c. 1770 - c. 1825 successfully challenged social hierarchies.

Outlines and Feedback

Thesis and outline practice submission 1.

Thesis: Although the revolutions in the Americas themselves were successful, they did not successfully challenge social hierarchies because the lower class people including the Black and mixed races and the slaves still dealt with prejudice and the Whites stayed in power. However, there was some successful social hierarchy change for the Whites because a new class of Whites did gain more political power.

Lower classes and Prejudice continued: Doc 2, Doc 4 (Only Creoles benefited, slavery continued) Doc 5

White Creoles gained power: Doc 3 (Bolivar was a Creole, Hidalgo and Mexican Revolution)

But, still great divisions among social classes–> underdevelopment + neocolonialism

(My thesis starts with “although” but that was not what I was trying to set up the complexity with. Would I still need to prove that the revolutions themselves were successful? If so, I would use doc 1 for that).

DBQ Teacher Feedback

A way to improve your thesis would be to indicate a specific revolution which corresponds to your description. As for your question, abut complexity, that nuance could  contribute  to complexity but that itself is not complexity. Overall your structure looks good to me, but for your first paragraph you could be a bit more narrow. Prejudice against the lower classes? Race or labor based prejudice? Narrow that down.

Thesis and Outline Practice Submission 2

Revolutions in the Americas that took place from 1770 - 1825 were generally extremely successful at challenging established social hierarchies, as they diminished the power of traditional elites and led to the liberation of formerly enslaved peoples, mainly African Americans and their descendants. However, In some areas such as Latin America, the new elites that arose from revolutions to replace the old, leaving the rest of the social hierarchy intact.

Diminished Power of Elites: Doc 5, Doc 3, American Revolution (Freedom from Monarchy), French Revolution

Freed Enslaved People: Doc 1, Doc 2, Haitian Revolution

Social Hierarchy Intact: Doc 4, Creole Revolutions (Creole replace Peninsulares)

Your thesis is sufficient. If you want to make it better you could consider using hedging and qualified language such as the types of hierarchies reinforced/challenged (Labor, race, class). Your essay structure looks fine, but if you are running out of time consider combining paragraphs 1 and 2. Hope this helps keep practicing.

Thesis and Outline Practice Submission 3

Thesis: Although revolutions ranging from 1770-1825, made a profound effect on social hierarchies at the time through freedom of slaves and the destruction of monarchies in Britain, they did not last for the long term in most regions such as Latin America as the continuance of racism towards people of dark skin and harsh as well as inconsiderate ruling dominated these new changes.

Freedom of slaves: 1, 2

Destruction of monarchies: 3

Continued racism: 4

Continuance of harsh/inconsiderate ruling: 5

Could my complexity have to do something with racism still persisting in modern-day or would that not give me the point?

This thesis does a good job setting up your claim (the word “profound” works well here) and your line of reasoning. It’s clear what you plan to develop for your argument in your body paragraphs.
As you practice with DBQs, consider how you might bring documents together in your body paragraphs. This outline works, but a stronger (and easier to write) outline might find ways to bring documents together under a common argument and keep your overall essay to just 2 body paragraphs.
Complexity isn’t about bringing in modern examples necessarily. That skill point is about developing a complex argument throughout the whole essay. Think about how you could connect all of your ideas together throughout the paragraphs and fully explain your ideas. That will get you closer on many points, whether or not it earns that one complexity point.

Thesis and Outline Practice Submission 4

Thesis: Although the revolutions in the Americas successfully overthrew the direct rule of the European monarchies and led to an emergence of creoles who came to power, social inequality still plague the lower class peoples of the newly liberated nations and the obstacles of forming a fair government amidst the grievances of the mixed race and indigenous people did not greatly contribute to change the preexisting social structures.

  • Social inequality: Docs 2,4; continued to enslave African Americans in North America
  • Obstacles of forming a fair of government: Doc 5; instability and poor governance=poverty and economy dependent on former colonizer countries
  • Creoles gaining power: Docs 1,3; Simon Bolivar, Jose de San Martin

And also, are there more than one definitive, and correct answer to this prompt? I’m worried about misinterpreting the documents on the exam, thus leading to a non-historically defensible essay. 

To start, I’ll address your question. Yes, there are multiple correct answers to this prompt (and any DBQ). The questions and document sets are designed to allow students to successfully defend any of a variety of claims. If your claim fits with your knowledge of history AND your understanding of the documents, it will likely count as “historically defensible.” (Basically, don’t stress it - you’ve got this!)
This is a really sophisticated thesis that addresses both political and social ideas. It looks like you know that to make the political/governance stuff work, you’ll need to connect it to social hierarchies. Good work.
I’m glad to see your body paragraphs include multiple documents. That’s the fastest/easiest way to be sure that you’re using them and actually developing an argument. Keep it up!

Fiveable

Stay Connected

© 2024 Fiveable Inc. All rights reserved.

AP® and SAT® are trademarks registered by the College Board, which is not affiliated with, and does not endorse this website.

WHAP Website Logo.png

Document-Based Question (DBQ)

What is a dbq, how to read the documents:, written documents, how to answer the prompt:, compare & contrast, cause & effect, change & continuity over time, how to earn all 7 points:.

Contextualization

Analysis & Sourcing

How to start writing the dbq, how to write a dbq:.

Attached below is a worksheet with an outline organizer for your DBQ. When practicing for your DBQ, feel free to download & print this to use:

iconfinder_10_171505 (1).png

7 documents

pngfind.com-documents-png-6607266 (1).pn

You are given 7 documents, and you are given a prompt, similar to an LEQ prompt. You need to write an essay, responding to the prompt, using evidence from the documents. ​You have 60 minutes in total, but of those 15 minutes are recommended for reading. The sections below describe the types of documents, types of prompts, and the rubric and how to earn each point. 

You are given 7 documents. The different types are described below:

Excerpt / Written Document

document-icon.png

Graphic, Diagram, Map, Cartoon

chart-646 (1).png

General Tips

Look at the sourcing before you read each doc to get an idea of what the doc might say

Write a quick summary (~3 bullet points) to summarize the content of each doc

Write a note of how each doc fits in with the prompt

Does it support or refute your thesis?

Which side of the prompt does it cover?

Which aspect (which body paragraph) of your prompt / thesis does it cover?

Any document with written paragraphs

Newspaper, letter, speech, historian's interpretation, constitution, religious text, etc.​

Special tips:

Before you read, read the sourcing & title and try to get an idea of what the doc might say

Take your time to understand the content of the doc; no need to rush​​

Write a few notes summarizing the doc

Figure out how the doc relates to the prompt

Does it argue one side or another?

Does it provide evidence for a specific geographical region?

Does it refute your thesis?

Which sub-category of the prompt does it answer?

Any document that is a photo

Any photo that a photographer might take, or an artist's depiction of a historical event

NOT a diagram, map, or something manmade or designed by historians

Read the sourcing & title to try to figure out what the photo might depict

Look at all aspects of the photo, get an idea of what it depicts

Does it represent a historical development?​

Does it represent an artistic movement?

Look for all signs of bias in the photo

Is it depicting a specific point of view?

Does it portray a certain culture as superior?

Does it portray a certain culture as inferior?​

Does it represent a military victory?​​​

This would mean one side is better than the other​

Does it portray something as bigger or exaggerated?

Means that the exaggerated thing is depicted as superior

Does it portray something as smaller?

Means that the thing that's depicted smaller is portrayed as inferior

Based on the point of view (bias) and the content, figure out how it relates to the prompt

Does it support / refute your thesis?

What aspect of the prompt does it answer?​

Any document that is a man-made photo

Graphic, diagram, political cartoon, map, etc.

Before you read, read the sourcing & title and try to get an idea of what the doc might depict

Look at the doc and try to figure out what it represents, or what topic it depicts

Think about the bias or point of view of the doc:

Does it represent the views of one side or another?

Is it depicting one side as exaggerated or superior to another?

Cartoons are generally biased

If it's a map, what is it representing?

Is it representing the map of industrial factories, trade routes, westward expansion, deciphered wind patterns, etc.?​

Once you figure this out, understand the historical context of the map

If it's a graphic or a diagram, what information does it detail?

If it's a population growth map, what allowed for population growth?​

Think of what the diagram depicts, and what allowed for that, and what's the historical context of that historical development?

Involves comparing & contrasting 2 different things

Most important thing is the argument: Not what the differences/similarities were, but HOW THEY WERE SIGNIFICANT

How to use the documents:

Some docs might explain the features of one of the comparand (the thing you compare), other docs may explain the other comparand

Some docs might cover both comparands

Figure out what the docs are saying for each comparand, and write your thesis based on that

What are they saying are similar & different about the 2?​

Involves examining what a certain historical development, and what were its causes & effects

What's more important is examining the significance of the causes, or how one cause/effect was more important than other causes/effects

Generally, 2 causes and 1 or 2 effects

Some docs might explain the event

Some docs might explain the causes, others might explain the effects

Draft a thesis based on the info about the causes & effects mentioned in the docs

Try to mention which causes were more significant than other causes

Involves examining what changed & what remained the same as a result of one event​

Some docs might explain the catalyst (the event that caused the changes/continuities) you write about

Some docs might describe the changes

Some docs might describe the continuities

Draft a thesis based on what the docs say about the changes & continuities

How to Earn all 7  Points:

Contextualization (1 point).

Examine the historical context of the story

Kind of like a "recap" or a "flashback"

Like at the beginning of a TV show, it shows a recap of the previous episode

How to write one

Always include the time period & possibly the location​

"In Europe in the period 1450 - 1750, ..."

Provide a brief 3-5 sentence recap of how the world arrived at the situation you are writing about in your essay

The contextualization should finish with how the world arrived at the historical development you write about in your thesis, so that there is a smooth transition from contextualization to the thesis

Sample Contextualizations

Topic: Related to the industrial revolution​

Before the 1750s, people were performing manual labor, making items by hand, which was very inefficient. From 1750-1900, Europe and the rest of the world underwent an economic transformation called the Industrial Revolution. Starting in Britain due to its abundance of raw materials & strong financial support, industrial capitalists built factories powered by waterwheels or coal that artificially produced goods such as textiles, eliminating the need to make them by hand. This brought a lot more people from the countryside to the cities, where they worked in factories for low wages. From Britain, the industrial revolution spread throughout Europe as well as to US, Egypt, Russia, and Japan. [Insert Thesis Here]

Topic: Related to imperialism​

In the period 1750-1900, Europe underwent an economic transformation known as the industrial revolution, where people would use artificial power to cheaply & efficiently manufacture goods in commercial factories in the cities, rather than making goods by hand at home. In order for these factories to produce goods, they needed raw materials, which is why they had to look to other nations like those in Africa and Asia to supply raw materials to them. Thi​s led to European imperialism, a development where Europeans started colonizing other nations throughout the world, especially in Africa and Asia, to establish export-oriented economies to get raw materials to supply to their factories. [Insert Thesis Here]

Thesis (1 point)

This is your argument

Must be something that can be opposed​

Someone else has to be able to write an essay whose thesis is the opposite of yours

Must contain an argument, and generally 2-3 examples (topics for body paragraphs)

Better to have a concession

Useful for complexity point

Format of Thesis & Examples

Color Key: 

Concession / Counterargument*

Similarities / Continuities / Causes

Difference for Comparand 1 / Changes / Effects

Differences for Comparand 2

*Concession is always optional. Described in the analysis section, it can be used to get the extra complexity point

Prompt: Compare & Contrast

Although some may believe [counterargument]* , w hile [comparand 1] and [comparand 2] both [insert similarities] , [comparand 1] was [insert difference for comparand 1], and [comparand 2] was [insert difference for comparand 2], which [is why / allowed for] [insert argument]. 

Although the Delhi Sultanate had very strict religious authority, while the Delhi Sultanate and the Chola Kingdom both used religion to maintain stability , the Delhi Sultanate was attempting to impose Islam on a Hindu-majority population , and the Chola Kingdom imposed Hindu on a Hindu population , which allowed for the Chola Kingdom to be more successful than the Delhi Sultanate. 

Prompt: Change & Continuity over Time

Although some may believe [counterargument]* , as a result of [catalyst],  while [continuity] stayed the same , [change] changed,   which [is why / allowed for] [insert argument]. 

Although some may believe the Catholic church actually became more powerful, as a result of the Protestant Reformation, while women still maintained strictly subordinate roles , there were more religious wars, and more monarchs were able to consolidate more power for themselves , which caused the Catholic church to decline in power.  

Prompt: Cause & Effect

Although some may believe [counterargument]* are the most important causes of [event] ,  [causes] were the main causes** , which caused [effects].  

Although some may believe that the desire to spread Christianity was the main cause of European imperialism , the desire to get raw materials and the need for more markets were the main causes , which led to a more integrated global economy and the development of technological infrastructure in the colonies. 

**Here, the argument is that the causes you described in the blue section are more important than the causes in the yellow section. There is no need for an extra argument at the end

Evidence (3 points)

This is where you put examples / pieces of evidence to support your thesis​

To get 1 point : Use evidence from 3 of the docs

To get 2 points : Use evidence from 6 of the docs, and put an extra analysis to connect it to the thesis

It's always better to use all 7 docs in case you use one incorrectly

To get 3 points : Use an extra piece of evidence (from your own knowledge, not from the docs), and put an extra analysis to connect it to the thesis

Examples of How to Write your Evidence

How to earn the first point:

To earn the 1st point, you need to describe / state evidence from 3 docs without connecting it to the thesis

According to document 3, the Chola Empire used Hinduism as the state religion. 

According to document 7, there were more factories in Britain than in France. 

How to earn the second point:

To earn the 2nd point, you need evidence from 6* docs, and you also need to connect it to the thesis

According to document 3, the Chola Empire used Hinduism as the state religion. Because the population was also mostly Hindu, the Chola Empire was able to maintain stability by using a common belief in Hinduism to stabilize its rule. 

According to document 7, there were more factories in Britain than in France. Thus, Britain had a larger industrial output than France, which is why it was able to manufacture more weapons during World War 1 and why France relied on Britain for support. 

*Always use all 7 docs to in case you use one doc incorrectly

How to earn the third point:

To earn the third point, you need to include one piece of evidence that is not in the documents and is from your own knowledge. 

Think of what evidence or what viewpoint is missing

If it's a compare & contrast: is there any other similarity or difference? Do you have any other evidence to support the topics of your thesis?

If it's a change & continuity over time: Is there any other evidence to support one of your changes or continuities?

If it's a cause & effect: Is there any other evidence or historical content that can support your causes or effects?

Analysis (2 points)

This is the hardest part

For 1 point, you need to explain how the source of 3 documents affects either your argument or what the document has to say

There are 4 parts of sourcing, and you ONLY NEED TO CHOOSE ONE

Explained in more detail below​

For the 2nd point, you need to use complex analysis in your argument

This is the most confusing

The easiest way is to weave a counterargument through your essay, which the concession already sets you up for

The best way is to not think about it too much and just put a bit more complex arguments into your essay rather than sticking to a strict format

Historical Context

Point of View

You need to choose ONE of the above and follow the instructions below. Each of the sections below has information about each aspect of sourcing. 

You need to do this for THREE different sources to earn full points (we recommend you do 4 in case one is wrong)

Historical Context:

Explain how the historical context of any document affects what the document argues

This document was written after WW1 when everyone was feeling depressed and economically poor, which explains why the priest is talking about a revival of religion and cheerful spirits. 

This document was written in a time after the Protestant Reformation when there were a lot of religious wars, which is why the document argues that Lutheranism is better than Calvinism. 

Explain how the intended audience of any document affects what the document argues

This document was written for the Armenians of the Ottoman empire, a Christian minority that was believed to conspire with the Allies, which is why the document is very aggressive toward them in asserting Ottoman dominance. 

This speech was written to the American people to gain support for the Treaty of Versailles, which is why it intends to boost nationalist sentiment and promote American power. 

Explain how the purpose of any document affects what the document argues

This speech was written by the Republic party with the purpose of convincing its audience to vote for them, which is why it argues that Free Silver, a democratic idea, is bad. 

This speech was written by John of Montecorvino, the Archbishop of Khanbaliq who sought to convert the Mongol boys to Christianity, which is why he emphasizes how Christianity allows one to achieve salvation. 

Point of View:

Explain how the point of view of any document affects what the document argues

This speech was written from the point of view of an Indian cotton farmer, which is why he writes that the British completely destroyed the Indian handmade textile industry. 

This document was written from the point of view of Grover Cleveland, an anti-imperialist president, which is why he writes about the harms of annexing Hawaii. 

The Complexity Point

The final point is the complexity point. This is given if you have a complex argument, and it is hard to achieve. The best way to think about this is do more than the prompt asks, and add a bit of extra analysis into the essay. 

The easiest way to do this is weave a counterargument through the essay. In our thesis samples above, we already set you up for this with our concession clause. 

How to Start Writing the DBQ:

First step is to outline your essay. Follow the steps below:

Read through each document, write a brief summary, and figure out how it relates to the prompt (which side/aspect does it argue?)​

Write your thesis. Write each aspect of the thesis (concession/counterargument, evidence 1, evidence 2, argument), and combine them

Write the outline for your body paragraphs. Write the topic for each body paragraph, and which docs you'll use in each. Also, denote where you'll use your outside evidence

Write an outline for your sourcing. Choose 4 different documents, and write the sourcing sentence following the guidelines in the sourcing section above

Start writing. Good luck!

PrepScholar

Choose Your Test

Sat / act prep online guides and tips, where to find the best dbq examples.

author image

Advanced Placement (AP)

feature-google-search-cc0-pixabay

One of the best ways to prepare for the DBQ (the "document-based question" on the AP European History, AP US History, and AP World History exams) is to look over sample questions and example essays. Doing this will help you to get a sense of what makes a good (and what makes a bad) DBQ response.

That said, not all DBQ essay examples are created equal. We'll briefly cover what makes a good DBQ example and then provide a list of example essays by course. Lastly, we'll give you some helpful tips on how to best use sample essays in your own preparation process.

What's a Good DBQ Example?

Without a doubt, the best sample resources come from the College Board . This is because they are the ones who design and administer the AP exams . This means the following:

Any DBQ essay example that the College Board provides will include a real DBQ prompt

All samples are real student responses from previous years , so you know they were written under the same conditions you'll have when you write your DBQ—in other words, they're authentic!

They not only have scores but also explanations of each essay's score , in accordance with the rubric

Each prompt includes several sample essays with a variety of scores

Some DBQ examples outside those available from the College Board might be worth looking at, particularly if they highlight how a particular essay could be improved. In general, though, a superior example will do the following:

Include the prompt and documents: It will be much easier for you to see how the information from the documents is integrated into the essay if you can actually look at the documents themselves!

Have a score: Seems simple, but you'd be surprised how many DBQ examples out there in the uncharted internet don't have one. Without a real, official score, it's hard to gauge how trustworthy a sample actually is.

With that in mind, I have compiled lists, organized by exam, of high-quality example DBQs below.

body-pencil-notes-cc0-pixabay

Don't spend all your study time on false starts with your practice DBQs. 

Every DBQ Example Essay You Could Ever Need, by Exam

Here are your example essays! We'll start with AP US History, then move to AP European History, and finally wrap up with AP World History.

AP US History: Official College Board Examples

The APUSH test was redesigned in 2015 and again in 2018, so right now there are eight  official College Board sets of sample essays you can use in your studies . Make sure to give yourself a 15-minute reading period and 45 minutes to write your answer. In addition, don't forget to use the current scoring guidelines when grading your own practice responses.

  • 2023 Free-Response Questions | Scoring Guidelines 2023
  • 2022 Free-Response Questions | Sample DBQ Responses 2022
  • 2021 Free-Response Questions | Sample DBQ Responses 2021
  • 2019 Free-Response Questions | Sample DBQ Responses 2019
  • 2018 Free-Response Questions | Sample DBQ Responses 2018
  • 2017 Free-Response Questions | Sample DBQ Responses 2017
  • 2016 Free-Response Questions | Sample DBQ Responses 2016
  • 2015 Free-Response Questions | Sample DBQ Responses 2015

If you want additional sample question sets, you can look at older College Board US History DBQ example response sets . To look at these, click "Free-Response Questions" for a given year. For the corresponding DBQ examples and scoring guidelines, click "Sample Responses Q1."

Note that these examples use the old rubric (which is integrated into the Scoring Guidelines for a given free-response section). General comments on the quality of the essay, outside information, and document analysis still apply, but the score is on a 9-point scale instead of the current 7-point scale, and some of the particulars will be different. Older DBQs had up to 12 documents, while the current format has seven documents.

If you do look at older DBQ examples, I recommend using the current rubric to re-grade the essays in the sample according to the 7-point scale. I'll also give more advice on how to use all these samples in your prep later on.

body-eagle-cc0-pixabay

Mr. Bald Eagle is an AP US History DBQ grader in his spare time.

AP European History: Official College Board Examples

Unfortunately, there aren't as many sample resources for the AP Euro DBQ compared to the other AP history tests because 2016 was the first year the AP Euro test was administered in the new format . Since then, more minor changes have been made in terms of time (you now have an hour on the DBQ) and individual parts of the rubric (you can view the current scoring guidelines here ).

This means there are seven sets of official samples graded with the current 7-point rubric:

The rest of the existing available samples were graded in the old 9-point format instead of the 7-point format implemented in 2016.

In the old format, there were 6 "core" points and 3 additional points possible. The old rubric is integrated with the sample responses for each question, but we'll highlight some key differences between the old and current formats :

With the old format, you were given a brief "historical background" section before the documents

There were more documents—up to 12—but the current format has seven

There was an emphasis on "grouping" the documents that is not present in the current rubric

There was also explicit emphasis on correctly interpreting the documents that is not found in the current rubric

While the essential components of the DBQ are still the same between the two test formats, you should definitely refer to the current rubric if you decide to look at any old AP European History samples . You might find it useful to look at old essays and score them in accordance with the current rubric.

Here are the old sample DBQ questions and essays, organized by year:

  • 2014 Free-Response Questions | Sample DBQ Responses 2014
  • 2013 Free-Response Questions | Sample DBQ Responses 2013
  • 2012 Free-Response Questions | Sample DBQ Responses 2012
  • 2011 Free-Response Questions | Sample DBQ Responses 2011

You can get samples in the old format all the way back to 1999 from the College Board . (Click "Free -Response Questions" for the questions and "Sample Response Q1" for the samples.)

body-castle-cc0-pixabay

Consider how you might integrate this castle into the DBQ that is your life.

AP World History: Official College Board Examples

The World History AP exam transitioned to a new format to more closely resemble AP US History and AP European History for the 2017 test. This means that there are six past exams available that use the current DBQ format:

Note that starting with the 2020 exam, AP World History will only cover the years 1200 to the present instead of thousands of years of history. As a result, both the course and exam have been renamed AP World History: Modern (a World History: Ancient course is in the works). What this means for you is that previous DBQs might have to do with time periods you're no longer required to study, so just keep this in mind.

In the old format, there were 7 "core" points and 2 additional points possible. The old rubric is integrated with the sample responses for each question, but we'll highlight some key differences between the old and current formats :

There were more documents—up to 10—but the current format has seven

There was an emphasis on "grouping" the documents on the old rubric that is not present in the current rubric

  • In the old rubric, you needed to identify one additional document that would aid in your analysis; the new rubric does not have this requirement

The essential components of the DBQ are still the same between the two formats, though you should definitely look at the current rubric if you study with any old AP World History questions and samples. You might find it useful to look at the old essays and score them according to the current rubric.

Here are old AP World History questions and DBQ sample responses , organized by year:

body-sphynx-cc0-pixabay

Don't worry, the old format isn't as old as this guy right here.

How Should I Use DBQ Examples to Prepare?

Now that you have all these examples, what should you do with them? In this section, we'll give you some tips on how to use example DBQs in your own AP history prep , including when to start using them and how many you should plan to review.

What Should I Do With These DBQs?

Official sample essay sets are a great way to test how well you understand the rubric. This is why we recommend that you grade a sample set early on in your study process—maybe even before you've written a practice DBQ .

Then, when you compare the scores you gave to the official scores and scoring notes given to the samples, you'll have a better idea of what parts of the rubric you don't really understand . If there are points you are consistently awarding differently than the graders, you’ll know those are skills you'll need to work on.

Keep giving points for the thesis and then finding out the sample didn't get those points? This tells you to work more on your thesis skills. Not giving points for historical context and then finding out the AP grader gave full credit? You need to work on recognizing what constitutes historical context according to the AP.

Check out my tips on building specific rubric-based skills in our guide on how to write a DBQ .

Once you've worked on some of those rubric skills you're weaker in, such as evaluating a good thesis or keeping track of how many documents were used, grade another sample set. This way you can see how your ability to grade the essays like an AP grader improves over time!

Obviously, grading sample exams is a much more difficult process if you're looking at examples in an old format. The old scores as awarded by the College Board will be helpful in establishing a ballpark —a 9 is still going to be a good essay using the current 7-point scale—but there may be some modest differences in grades between the two scales. (For example, maybe that perfect 9 is now more like a 6 out of 7 due to rubric changes.)

For practice grading with old samples, you might want to pull out two copies of the current rubric, recruit a trusted study buddy or academic advisor (or even two study buddies!), and have each of you re-grade the samples .

You can then discuss any major differences in the grades each of you awarded. Having multiple sets of eyes will help you determine whether the scores you're giving are reasonable, since you won’t have an official 7-point College Board score for comparison.

Looking for help studying for your AP exam? Our one-on-one online AP tutoring services can help you prepare for your AP exams. Get matched with a top tutor who got a high score on the exam you're studying for!

How Many Example DBQs Should I Be Using?

The answer to this question depends on your study plans.

If it's six months before the exam and you plan on transforming yourself into a hard diamond of DBQ excellence, you might do practice grading on a sample set every few weeks to a month to check your progress to being able to think like an AP grader. In this case, you would probably use six to nine official sample sets.

If, on the other hand, the exam is in a month and you're just trying to get in some extra skill-polishing, you might do a sample set every week to 10 days . It makes sense to check your skills more often when you have less time to study because you want to be sure that you are focusing your time on the skills that need the most work. For a short time frame, expect to use somewhere in the range of three to four official sample sets.

Either way, you should be integrating your sample essay grading with skills practice and doing some practice DBQ writing of your own .

Toward the end of your study time, you could even integrate DBQ writing practice with sample grading. Read and complete a timed prompt and then grade the sample set for that prompt, including yours! The other essays will help give you a sense of what score your essay might have received that year and any areas you might have overlooked.

There's no one-size-fits-all approach to using sample sets, but in general they are a useful tool for making sure you have a good idea what the DBQ graders will be looking for when you write your own DBQ on test day.

body-ostriches-cc0-pixabay

Hey, where can we find a good DBQ around here?

Closing Thoughts: Example DBQs for AP History Tests

Example DBQ essays are a valuable resource in your arsenal of study strategies for the AP history exams. Grading samples carefully will help you get a sense of your own blind spots so you'll know what skills to focus on in your prep.

That said, sample essays will be most useful when integrated with your own targeted skills prep . Grading 100 sample essays won't help you if you aren't practicing your skills; rather, you'll just keep making the same mistakes over and over again.

Make sure you aren't using sample essays to avoid writing practice DBQs either—you'll want to do at least a couple, even if you only have a month to practice.

And there you have it, folks. With this list of DBQ examples and tips on how to use them, you are all prepared to integrate samples into your study strategy!

body-whats-next-cc0-pixabay

What's Next?

Still not sure what a DBQ is? Check out my explanation of the DBQ to learn the basics.

Want tips on how to really dig in and study for AP history tests? We've got a complete how-to guide on preparing for and writing the DBQ .

If you're still studying for AP World History, check out our top AP World History study guide , or get more practice tests from our complete list .

Want more study material for AP US History? Look into this article on the best notes to use for studying from one of our experts. Also, read our review of the best AP US History textbooks !

Want to improve your SAT score by 160 points or your ACT score by 4 points?   We've written a guide for each test about the top 5 strategies you must be using to have a shot at improving your score. Download them for free now:

Ellen has extensive education mentorship experience and is deeply committed to helping students succeed in all areas of life. She received a BA from Harvard in Folklore and Mythology and is currently pursuing graduate studies at Columbia University.

Ask a Question Below

Have any questions about this article or other topics? Ask below and we'll reply!

Improve With Our Famous Guides

  • For All Students

The 5 Strategies You Must Be Using to Improve 160+ SAT Points

How to Get a Perfect 1600, by a Perfect Scorer

Series: How to Get 800 on Each SAT Section:

Score 800 on SAT Math

Score 800 on SAT Reading

Score 800 on SAT Writing

Series: How to Get to 600 on Each SAT Section:

Score 600 on SAT Math

Score 600 on SAT Reading

Score 600 on SAT Writing

Free Complete Official SAT Practice Tests

What SAT Target Score Should You Be Aiming For?

15 Strategies to Improve Your SAT Essay

The 5 Strategies You Must Be Using to Improve 4+ ACT Points

How to Get a Perfect 36 ACT, by a Perfect Scorer

Series: How to Get 36 on Each ACT Section:

36 on ACT English

36 on ACT Math

36 on ACT Reading

36 on ACT Science

Series: How to Get to 24 on Each ACT Section:

24 on ACT English

24 on ACT Math

24 on ACT Reading

24 on ACT Science

What ACT target score should you be aiming for?

ACT Vocabulary You Must Know

ACT Writing: 15 Tips to Raise Your Essay Score

How to Get Into Harvard and the Ivy League

How to Get a Perfect 4.0 GPA

How to Write an Amazing College Essay

What Exactly Are Colleges Looking For?

Is the ACT easier than the SAT? A Comprehensive Guide

Should you retake your SAT or ACT?

When should you take the SAT or ACT?

Stay Informed

Get the latest articles and test prep tips!

Follow us on Facebook (icon)

Looking for Graduate School Test Prep?

Check out our top-rated graduate blogs here:

GRE Online Prep Blog

GMAT Online Prep Blog

TOEFL Online Prep Blog

Holly R. "I am absolutely overjoyed and cannot thank you enough for helping me!”

The Magoosh logo is the word Magoosh spelled with each letter o replaced with a check mark in a circle.

APUSH Document Based Questions and Responses: A Study Guide

Document Based Questions tend to freak students out the most on the APUSH exam. This is understandable; not only do you have to read documents, you have to write a coherent essay about them.

However, we’ve got you covered here at Magoosh. For a step-by-step breakdown of what you need to do to write a Document Based Question (DBQ), check out my other blog posts on 3 Steps to a DBQ Essay that Works and How to Write a DBQ Essay . This post, though, will focus on what other students have done to make their Document Based Question essays successful.

I will take you through one DBQ on a prior APUSH exam and give you the ins and outs, and the dos and don’ts. At the end, I will provide a link to a DBQ essay for you to practice some of the things that successful test-takers have done. There, you can compare your essay to the scoring notes provided by College Board. In fact, everything I present on this post will be provided by College Board – you can (and should!) check out their website for more tips.

Sound good? Let’s go!

Document Based Question #1

This is taken from the 2016 APUSH Exam . The DBQ for this section asks you to do the following:

Explain the causes of the rise of a women’s rights movement in the period 1940–1975.

You will have 55 minutes to answer that question. The College Board suggests 15 minutes for reading and 40 minutes for writing, although if you are a fast and careful reader, you can start writing before your 15-minute reading period is done.

I won’t post all of the documents that you have to reference (there are 7 after all!), but the following two documents are representative of the types of documents you will encounter on a DBQ.

thesis statement dbq example

As you can see, there’s a mix of photographs, advertisements, and text that you will be expected to incorporate into your essay.

Still with me? Good. Next, let’s look into what an essay should have in it.

Scoring Notes for Document Based Questions

The following (including descriptions) comes straight from the APUSH scoring notes. I’ll break down parts of it later to make sure that you understand what they want to see.

Your DBQ essay should have the following (for a maximum of 7 points):

  • Thesis: Present a thesis that makes a historically defensible claim and responds to all parts of the question. The thesis must consist of one or more sentences located in one place, either in the introduction or the conclusion.
  • Argument Development: Develop and support a cohesive argument that recognizes and accounts for historical complexity by explicitly illustrating relationships among historical evidence such as contradiction, corroboration, and/or qualification.
  • Use of the Documents: Utilize the content of at least six of the documents to support the stated thesis or a relevant argument.
  • Sourcing the Documents: Explain the significance of the author’s point of view, author’s purpose, historical context, and/or audience for at least four documents.
  • Contextualization: Situate the argument by explaining the broader historical events, developments, or processes immediately relevant to the question.
  • Outside Evidence: Provide an example or additional piece of specific evidence beyond those found in the documents to support or qualify the argument.
  • A development in a different historical period, situation, era, or geographical area.
  • A course theme and/or approach to history that is not the focus of the essay (such as political, economic, social, cultural, or intellectual history).

Yes, it’s a lot. But students have done it before, and so can you! Just because of the limits of space, I am only going to show you what to do – and what not to do – on the parts where I have seen students struggle most: thesis and synthesis.

Thesis Statements for Document Based Questions

Your thesis statement is the bread and butter of any essay you write for the APUSH exam. As I have stated before in previous posts, you should spend the most time on your thesis because a strong thesis will guide the rest of your essay.

But what makes a thesis strong? As mysterious as that question may seem, it is relatively straightforward:

A strong thesis directly answers the question being asked by referencing specific times, movements, or ideas.

It’s that simple! Well, it’s sort of simple. Developing a strong thesis is hard work, but let’s begin at the beginning. Here’s the question being asked: Explain the causes of the rise of a women’s rights movement in the period 1940–1975.

Notice that the question asks for causes, meaning MORE than one. Also, notice that the question gives you a defined time period to work with. Therefore, your thesis shouldn’t deal with any events, ideas, or people outside of that time period.

Let’s look at two student examples.

Example Thesis #1:

The women’s rights movement arose as a result of women’s experiences with inequality at work and the influence of other rights movements.

Does the student directly answer the question being asked? Yes. According to the student, the women’s rights movement was caused by the experiences of women dealing with inequality at work and the influence of other rights movements in the same time.

Is the student being specific? Yes. I know that this student will be organizing their essay in two big chunks: inequality in the workforce and civil rights movements.

Notice that this student didn’t give the longest answer possible, and the response was not necessarily the most eloquent, but that student still got a point for their thesis.

Example Thesis #2:

The woman’s rights movement was the product of unfair treatment in economics, politics, and society.

Does the student directly answer the question being asked? Well, yes, but I am unclear what “economics, politics, and society” means.

Is the student being specific? Not at all. There could be thousands of things that go under economics, politics, and society – and many things could be considered “unfair” – so I have no idea what the student will be arguing in this DBQ.

Thesis Statement Dos and Don’ts

  • Directly answer the question being asked.
  • Be specific.
  • Write a thesis statement like the first example.

Don’t:

  • Answer the question in a confusing way or answer some other question you think the test SHOULD be asking.
  • Be general.

Synthesis in Document Based Questions

This is a newer component of the DBQ. You need to demonstrate your understanding of history by being able to go beyond the documents they provide you and make connections between different parts of history. This does NOT mean that you need to spend all of your time racking your brain for more evidence. However, it does mean that you should have a solid understanding of US History and can extend your argument to other time periods or themes. Let’s look at some student examples to explain what I mean.

Student Example #1:

The conditions that helped cause the rise of the women’s rights movement in the 20th century were similar to those that helped cause the rise of a movement for greater women’s rights in the 1840s. In both periods, calls for greater rights for African Americans led women to demand more of a voice in social and political reforms.

This student explains that the conditions for women’s rights movements were similar in two different time periods, extending the argument beyond this one moment in US history.

Student Example #2:

A development in a different historical period was when Alice Paul went on hunger strikes and protests in from of the White House to gain attention on passing an amendment that would give women their rights. Paul’s fight for women’s rights started with trying to get equal voting rights for women. This links to how in 1940-1975 women were fighting for equal rights in wages and other important rights.

This student makes a connection to another time period by arguing that the fight for equal rights did not begin in 1940; instead, women had been active for some time in US history to achieve equal rights.

Student Example #3:

The Seneca Falls convention also served to help inspire women around the world to gain equal rights. The speech given clearly stated the way things were being conducted was unconstitutional and women should not be socially inferior to men.

Unlike the first two student responses, this third response does NOT connect back to the time period in the question. I am unclear from this student response whether the connection is gaining equal rights for women, the persistence of inequality, or the changing interpretations of the Constitution. This student did not receive a point for the synthesis criteria.

Synthesis Dos and Don’ts

  • Connect back to the time period of the question.
  • State something that you feel is an “obvious” connection, but never make a connection yourself. You should be doing that work for your reader.

Document Based Question #2: Your Turn!

Although I haven’t outlined every single component of the DBQ, you should look at the two blog posts I linked to at the beginning of this article for more references.

But now it’s time for you to dive in! You will only get better by practicing.

You should practice with the 2015 Document Based Question 1 . In that document, you will have access to the questions and sample student responses.

Good luck, and let me know how it goes!

Allena Berry

Allena Berry loves history; that should be known upfront. She loves it so much that she not only taught high school history and psychology after receiving her Master’s degree at Stanford University, she is now studying how students learn history at Northwestern. That being said, she does not have a favorite historical time period (so don’t bother asking). In addition to history, she enjoys writing, practicing yoga, and scouring Craigslist for her next DIY project or midcentury modern piece of furniture.

View all posts

More from Magoosh

APUSH Short Answer Questions and Responses: A Study Guide

Leave a Reply Cancel reply

Your email address will not be published. Required fields are marked *

  • PRO Courses Guides New Tech Help Pro Expert Videos About wikiHow Pro Upgrade Sign In
  • EDIT Edit this Article
  • EXPLORE Tech Help Pro About Us Random Article Quizzes Request a New Article Community Dashboard This Or That Game Popular Categories Arts and Entertainment Artwork Books Movies Computers and Electronics Computers Phone Skills Technology Hacks Health Men's Health Mental Health Women's Health Relationships Dating Love Relationship Issues Hobbies and Crafts Crafts Drawing Games Education & Communication Communication Skills Personal Development Studying Personal Care and Style Fashion Hair Care Personal Hygiene Youth Personal Care School Stuff Dating All Categories Arts and Entertainment Finance and Business Home and Garden Relationship Quizzes Cars & Other Vehicles Food and Entertaining Personal Care and Style Sports and Fitness Computers and Electronics Health Pets and Animals Travel Education & Communication Hobbies and Crafts Philosophy and Religion Work World Family Life Holidays and Traditions Relationships Youth
  • Browse Articles
  • Learn Something New
  • Quizzes Hot
  • This Or That Game
  • Train Your Brain
  • Explore More
  • Support wikiHow
  • About wikiHow
  • Log in / Sign up
  • Education and Communications
  • College University and Postgraduate
  • Academic Writing

How to Write a DBQ Essay

Last Updated: February 27, 2024 Fact Checked

This article was co-authored by Emily Listmann, MA . Emily Listmann is a Private Tutor and Life Coach in Santa Cruz, California. In 2018, she founded Mindful & Well, a natural healing and wellness coaching service. She has worked as a Social Studies Teacher, Curriculum Coordinator, and an SAT Prep Teacher. She received her MA in Education from the Stanford Graduate School of Education in 2014. Emily also received her Wellness Coach Certificate from Cornell University and completed the Mindfulness Training by Mindful Schools. This article has been fact-checked, ensuring the accuracy of any cited facts and confirming the authority of its sources. This article has been viewed 681,452 times.

In the past, Document Based Questions (DBQ) were rarely found outside of AP history exams. However, they’re now used in social studies classes across grade levels, so you’re bound to take a DBQ test at some point. [1] X Research source Going into the test, you will need strong background knowledge of the time periods and geographical areas on which you will be tested. Your documents will always relate back directly to the major subjects and themes of your class. The key to success is to analyze the provided documents and use them to support an argument in response to the essay prompt. While DBQ tests are rigorous, they allow you to actually do historical work instead of merely memorize facts. Don’t stress, put on your historian hat, and start investigating!

Writing Help

thesis statement dbq example

Analyzing the Documents

Step 1 Review the documents for 10 to 15 minutes.

  • For an AP exam, you’ll then have 45 minutes to write your essay. Exact times may vary for other exams and assignments but, for all DBQ essays, document analysis is the first step.
  • For an AP exam, you will also need to include a thesis, set the prompt’s historical context, use 6 documents to support an argument, describe 1 piece of outside evidence, and discuss the point of view or context of at least 3 of the sources. Label these elements as you review and outline so you don’t forget something.

Step 2 Identify the prompt’s keywords and assigned tasks.

  • A prompt might ask you to analyze or explain the causes of a historical development, such as, “Explain how the Progressive Movement gained social, political, and cultural influence from the 1890s to the 1920s in the United States.”
  • You might need to use primary sources to compare and contrast differing attitudes or points of view toward a concept, policy, or event, such as, “Compare and contrast the differing attitudes towards women’s rights in the United States from 1890 to 1920.”
  • Keywords in these examples inform you how to read your sources. For instance, to compare and contrast differing attitudes, you’ll need to identify your sources’ authors, categorize their points of view, and figure out how attitudes changed over the specified period of time.

Step 3 Note your documents’ authors, points of view, and other details.

  • Suppose one of the documents is a suffragette’s diary entry. Passages in the entry that detail her advocacy for the Women’s Rights Movement are evidence of her point of view. In contrast, another document is newspaper article written around the same time that opposes suffrage.
  • A diary entry might not have an intended audience but, for documents such as letters, pamphlets, and newspaper articles, you’ll need to identify the author’s likely readers.
  • Most of your sources will probably be written documents, but you’ll likely encounter political cartoons, photographs, maps, or graphs. The U.S. Library of Congress offers a helpful guide to reading specific primary source categories at https://www.loc.gov/teachers/usingprimarysources/guides.html .

Step 4 Place your sources into categories based on the essay prompt.

  • Suppose you have a letter sent from one suffragette to another about the methods used to obtain the right to vote. This document may help you infer how attitudes vary among the movement’s supporters.
  • A newspaper article depicting suffragettes as unpatriotic women who would sabotage World War I for the United States helps you understand the opposing attitude.
  • Perhaps other sources include a 1917 editorial on the harsh treatment of imprisoned suffragists and an article on major political endorsements for women’s suffrage. From these, you’d infer that 1917 marked a pivotal year, and that the role women played on the home front during World War I would lead to broader support for suffrage.

Step 5 Think of relevant outside information to include in your essay.

  • For instance, perhaps you read that the National American Woman Suffrage association (NAWSA) made a strategic shift in 1916 from focusing on state-by-state suffrage to prioritizing a constitutional amendment. Mentioning this switch to a more aggressive strategy supports your claim that the stage was set for a 1917 turning point in popular support for women’s suffrage.
  • When you think of outside evidence during the planning stages, jot it down so you can refer to it when you write your essay. A good spot could be in the margin of a document that relates to the outside information.

Developing an Argument

Step 1 Review the prompt and form a perspective after reading the documents.

  • For example, after reviewing the documents related to women’s suffrage, identify the opposing attitudes, how they differed, and how they changed over time.
  • Your rough argument at this stage could be, “Those in opposition saw suffragettes as unpatriotic and unfeminine. Attitudes within the suffrage movement were divided between conservative and confrontational elements. By the end of World War I, changing perceptions of the role of women contributed to growing popular support for suffrage.”

Step 2 Refine your rough...

  • Suppose your DBQ is, “How did World War I affect attitudes toward women’s suffrage in the United States?” A strong tentative thesis would be, “The roles women played in the workforce and in support of the war effort contributed to growing popular support for the suffrage movement.”
  • A weak thesis would be, “World War I affected how Americans perceived women’s suffrage.” This simply restates the prompt.

Step 3 Make an outline of your argument’s structure.

  • For example, under numeral I., write, “New Woman: perceptions shift in the 1890s.” This section will explain the 1890s concept of the New Woman, which rejected traditional characterizations of women as dependent and fragile. You’ll argue that this, in part, set the stage for shifting attitudes during and following World War I.
  • You can start your planning your essay during the reading portion of the test. If necessary, take around 5 minutes out of the writing portion to finish outlining your argument.

Step 4 Plug your document citations into the outline.

  • For instance, under “I. New Woman: perceptions shift in the 1890s,” write “(Doc 1),” which is a pamphlet praising women who ride bicycles, which was seen as “unladylike” at the time.
  • Beneath that line, write “(Doc 2),” which is an article that defends the traditional view that women should remain in the household. You’ll use this document to explain the opposing views that set the context for suffrage debates in the 1900s and 1910s.

Step 5 Refine your thesis after making the outline.

  • Suppose your tentative thesis is, “The roles women played in the workforce and in support of the war effort contributed to growing popular support for the suffrage movement.” You decide that “contributed” isn’t strong enough, and swap it out for “led” to emphasize causation.

Drafting Your Essay

Step 1 Keep your eye on the clock and plan your time strategically.

  • If you have 45 minutes to write, take about 5 minutes to make an outline. If you have an introduction, 3 main points that cite 6 documents, and a conclusion, plan on spending 7 minutes or less on each of these 5 sections. That will leave you 5 minutes to proofread or to serve as a buffer in case you need more time.
  • Check the time periodically as you write to ensure you’re staying on target.

Step 2 Include your thesis and 1 to 2 sentences of context in your introduction.

  • To set the context, you might write, “The Progressive Era, which spanned roughly from 1890 to 1920, was a time of political, economic, and cultural reform in the United States. A central movement of the era, the Women’s Rights Movement gained momentum as perceptions of the role of women dramatically shifted.”
  • If you’d prefer to get straight to the point, feel free to start your introduction with your thesis, then set the context.
  • A timed DBQ essay test doesn’t leave you much time to write a long introduction, so get straight to analyzing the documents rather than spell out a long, detailed intro.

Step 3 Write your body paragraphs.

  • Each body section should have a topic sentence to let the reader know you’re transitioning to a new piece of evidence. For example, start the first section with, “The 1890s saw shifts in perception that set the stage for the major advances in women’s suffrage during and following World War I.”
  • Be sure to cite your documents to support each part of your argument. Include direct quotes sparingly, if at all, and prioritize analysis of a source over merely quoting it.
  • Whenever you mention a document or information within a document, add parentheses and the number of the document at the end of the sentence, like this: “Women who were not suffragettes but still supported the movement wrote letters discussing their desire to help (Document 2).”

Step 4 Make sure to show how each body paragraph connects to your thesis.

  • For example, a private diary entry from 1916 dismissing suffrage as morally corrupt isn’t necessarily a reflection of broader public opinion. There's more to consider than just its content, or what it says.
  • Suppose a more reliable document, such as a major newspaper article on the 1916 Democratic and Republican national conventions, details the growing political and public support for women’s suffrage. You’d use this source to show that the diary entry conveys an attitude that was becoming less popular.

Step 5 Weave together your argument in your conclusion.

  • In your essay on World War I and women’s suffrage, you could summarize your argument, then mention that the war similarly impacted women’s voting rights on an international scale.

Revising Your Draft

Step 1 Proofread your essay for spelling and grammatical mistakes.

  • If you’re taking an AP history exam or other timed test, minor errors are acceptable as long as they don't affect your argument. Spelling mistakes, for instance, won’t result in a loss of points if the scorer can still understand the word, such as “sufrage” instead of “suffrage.”

Step 2 Make sure you’ve included all required elements.

  • A clear thesis statement.
  • Set the prompt’s broader historical context.
  • Support your argument using 6 of the 7 included documents.
  • Identify and explain 1 piece of historical evidence other than the included documents.
  • Describe 3 of the documents’ points of view, purposes, audiences, or context.
  • Demonstrate a complex understanding of the topic, such as by discussing causation, change, continuity, or connections to other historical periods.

Step 3 Check that your names, dates, and other facts are accurate.

  • As with spelling and grammar, minor errors are acceptable as long as the scorer knows what you mean. Little spelling mistakes are fine, but you’ll lose points if you write that a source supports suffrage when it doesn’t.

Community Q&A

wikiHow Staff Editor

  • Remember that you shouldn't just identify or summarize a document. Explain why a source is important, and tie each reference into your argument. Thanks Helpful 0 Not Helpful 0
  • If you’re taking an AP history exam, find exam rubrics, practice tests, and other resources at https://apcentral.collegeboard.org/courses . Thanks Helpful 0 Not Helpful 0
  • Taking a timed test can be tough, so time yourself when you take practice tests. Thanks Helpful 0 Not Helpful 0

thesis statement dbq example

You Might Also Like

Write a Compare and Contrast Essay

  • ↑ http://www.gpb.org/blogs/education-matters/2016/10/14/getting-started-document-based-questions
  • ↑ https://sourceessay.com/tips-to-write-an-impressive-dbq-essay/
  • ↑ https://libguides.jcu.edu.au/writing/writing1
  • ↑ https://apcentral.collegeboard.org/pdf/ap-us-history-dbq-2018.pdf?course=ap-united-states-history
  • ↑ https://apcentral.collegeboard.org/pdf/ap-us-history-course-and-exam-description.pdf
  • ↑ https://writingcenter.unc.edu/tips-and-tools/editing-and-proofreading/

About This Article

Emily Listmann, MA

Document-Based Questions, or DBQ essays, are often used in social studies classes to test your ability to do historical work rather than simply memorize facts. Start by spending some time reviewing the documents and developing an argument. Pay special attention to keywords in the prompt that will help you construct your argument. For example, if the prompt includes the words "compare and contrast," you'll need to include 2 different viewpoints in your essay and compare them. Then, as you read your sources, note the authors, points of view, and other key details that will help you figure out how to use the documents. Once you’ve reviewed all of the material, come up with your response. Sketch out a tentative thesis that encapsulates your argument and make an outline for your essay. You can then draft your essay, starting with an introduction that gives context and states your thesis, followed by supporting body paragraphs. To learn how to write a conclusion for your DBQ, keep reading! Did this summary help you? Yes No

  • Send fan mail to authors

Reader Success Stories

Livi G.

Apr 4, 2017

Did this article help you?

thesis statement dbq example

Emily Balint

Apr 18, 2016

Miracle Frappe

Miracle Frappe

May 6, 2019

Seorae Kim

Oct 30, 2016

Kate Alberry

Kate Alberry

Dec 30, 2020

Am I Smart Quiz

Featured Articles

Reduce Acne Scars with Home Remedies

Trending Articles

How to Do Fourth of July Nails: 40+ Nail Art Ideas

Watch Articles

Make Stamped Metal Jewelry

  • Terms of Use
  • Privacy Policy
  • Do Not Sell or Share My Info
  • Not Selling Info

Don’t miss out! Sign up for

wikiHow’s newsletter

IB CRASH COURSE FOR MAY SESSION 2024

For more details related to IBDP 1 Crash Course, Please Download IBDP 1 Brochure. For more details related to IBDP 2 Crash Course, Please Download IBDP 2 Brochure. For more details related to IBMYP Crash Course, Please Download IBMYP Brochure.

For Any Queries related to crash course, Please call at +918825012255

thesis statement dbq example

DBQ Example: How to Tackle Document-Based Questions

  • September 6, 2023
  • General Information

DBQ Example

Table of Contents

  • 1 What is a DBQ?
  • 2 How to effectively answer a DBQ
  • 3 Tips for tackling DBQs
  • 4 An example of a DBQ question
  • 5 How to score well on DBQs
  • 6 Conclusion

Are you ready to tackle document-based questions (DBQs) with confidence? Whether you’re a student working on an assignment or preparing for an exam, mastering the art of answering DBQs is essential. These complex questions require critical thinking, analysis, and the ability to synthesize information from various documents. But fear not! In this blog post, we’ll guide you through the process of effectively tackling DBQs by providing tips and strategies that will help you shine in any situation. So grab your pen and get ready to dive into a real-life DBQ example as we unlock the secrets to success!

What is a DBQ?

A Document-Based Question, commonly known as a DBQ, is an assessment tool used in history and social studies classes to evaluate students’ ability to analyze and interpret primary source documents. These documents can include letters, photographs, speeches, maps, and other artifacts from a particular historical period.

DBQs are designed to test your critical thinking skills by requiring you to examine the provided documents and develop an argument or response based on the information they contain. This means that you need to not only understand the content of each document but also identify any biases or perspectives presented.

The goal of a DBQ is not simply to regurgitate facts from the documents but rather to demonstrate your understanding of historical context and your ability to construct a well-supported argument using evidence from the sources.

To successfully tackle a DBQ, it’s important to approach it with a clear plan in mind. Start by carefully reading through all of the provided documents and taking notes on key details or themes that emerge. Then, consider how these pieces fit together within the larger historical context being examined.

As you begin crafting your response, remember that organization is key. Your essay should have a clear introduction stating your thesis statement – this will be your main argument or claim based on analyzing the given materials. Each subsequent paragraph should focus on supporting evidence from specific documents while also addressing counterarguments or alternative viewpoints.

By effectively tackling DBQs using these strategies outlined above—and practicing them consistently—you’ll soon find yourself confidently navigating through complex historical questions like never before! So let’s dive into some practical tips for success in our next section!

How to effectively answer a DBQ

When it comes to tackling a Document-Based Question (DBQ), it’s important to have a clear and effective strategy in place. Here are some tips to help you navigate through the process and come up with a strong response.

Make sure you thoroughly read and understand the prompt. Take note of any specific instructions or guidelines provided. This will help you focus your analysis and avoid going off track.

Next, carefully examine each document provided. Pay attention to details such as the author, date, and context of each source. Look for patterns or themes that emerge across multiple documents, as this can provide valuable insights for your response.

As you analyze the documents, be sure to consider their point of view or bias. Consider why the author may have written what they did and how this influences their perspective on the topic at hand.

When crafting your response, be sure to incorporate evidence from the documents into your argument. Use direct quotes or specific examples to support your points and demonstrate your understanding of the material.

Additionally, don’t forget about outside information! While the primary focus should be on analyzing the provided documents, incorporating relevant knowledge from outside sources can strengthen your argument and showcase a deeper understanding of the topic.

Take time to organize your thoughts before writing out your response. Create an outline or structure for your essay that clearly lays out how you plan to address each part of the prompt. This will ensure that your ideas flow logically and cohesively throughout.

By following these strategies for effectively answering DBQs, you’ll be well-equipped to tackle any question that comes your way! Remember: practice makes perfect when it comes to mastering this format.

Also Read:  Epilogue Definition: Examining the Final Chapters and Closure in Literary Works

Tips for tackling DBQs

1. Analyze the Prompt: Before diving into the documents, take a moment to carefully read and understand the prompt. Pay attention to keywords and directives that guide your response.

2. Familiarize Yourself with the Documents: Take a quick scan of all the provided documents to get an overview of their content. Note any patterns or themes that emerge.

3. Create a Thesis Statement: Craft a clear and concise thesis statement that directly addresses the prompt and outlines your main argument. This will serve as a roadmap for your essay.

4. Group Documents Strategically: As you analyze each document, group them based on similarities in theme or perspective. This helps to organize your thoughts and create coherent paragraphs.

5. Use Outside Knowledge: To strengthen your analysis, bring in relevant knowledge from outside sources when appropriate. This shows depth of understanding and can earn you extra points.

6. Cite Specific Evidence: When referencing specific documents or outside information, be sure to cite evidence using parenthetical citations (e.g., Document 1) or by mentioning key details.

7. Practice Time Management: Be mindful of time constraints when tackling DBQs during exams or timed assignments. Break down your essay into manageable sections to ensure you have enough time for each component.

8. Avoid Summarizing Documents: Instead of simply summarizing what each document says, focus on analyzing its significance within the context of the prompt and supporting your thesis statement.

9. Revise Your Essay: After completing your initial draft, take some time to review and revise it for clarity, coherence, grammar errors,and punctuation mistakes . Make sure each paragraph builds upon one another smoothly.

10. Seek Feedback if Possible:: If possible , ask someone knowledgeable about DBQs or an instructor familiar with this type of writing assignment,,to provide feedback on how well you addressed the prompt,cited evidence,and organizedyour ideas.

An example of a DBQ question

Let’s dive into an example of a Document-Based Question (DBQ) to get a better understanding of how they work. Imagine you’re given the following prompt: “Evaluate the impact of industrialization on workers in the late 19th and early 20th centuries.”

To effectively tackle this question, you’ll need to analyze and interpret a combination of primary and secondary sources. These documents could include photographs, letters, newspaper articles, or even excerpts from historical texts.

As you examine each document, pay attention to the author’s perspective and purpose. Are they advocating for workers’ rights or promoting capitalism? Look for patterns, contradictions, and connections between different sources.

Remember that your response should be well-structured with an introduction that clearly states your thesis statement. Each body paragraph should focus on analyzing specific documents while incorporating outside knowledge when relevant.

When crafting your essay, make sure to use evidence from the provided documents to support your arguments. Your conclusion should summarize your main points without introducing any new information.

By following these steps and practicing with sample DBQs beforehand, you’ll become more comfortable tackling these complex questions!

How to score well on DBQs

Scoring well on Document-Based Questions (DBQs) can be a challenge, but with the right approach, you can ace these exams and showcase your historical analysis skills. Here are some tips to help you score well on DBQs:

1. Understand the prompt: Carefully read and analyze the prompt before diving into the documents. Identify key terms and concepts that will guide your response.

2. Analyze the documents: Take time to thoroughly examine each document provided. Look for patterns, biases, and connections between different sources. Consider their context, authorship, purpose, and audience.

3. Organize your thoughts: Before writing your essay, create an outline or graphic organizer to structure your arguments. This will ensure a logical flow of ideas and help you stay focused throughout.

4. Use evidence effectively: Support your claims with specific examples from the documents provided. Be sure to explain how each piece of evidence supports your argument.

5. Provide historical context: Situate your analysis within its historical context by referencing relevant events or developments outside of the given documents.

6. Practice good writing skills: Clearly communicate your ideas in a concise and coherent manner using proper grammar and vocabulary appropriate for an academic setting.

7. Don’t forget about counterarguments: Acknowledge other perspectives or interpretations when appropriate and address them in order to strengthen your own argument.

By following these strategies, you’ll be able to approach DBQs with confidence and increase your chances of scoring well on these challenging assessments. Good luck!

Also Read:  DBQ Meaning: Decoding the Purpose and Approach of Document-Based Questions

Mastering the art of tackling Document-Based Questions (DBQs) is crucial for success in history and social science exams. By understanding what a DBQ entails, effectively answering the question, utilizing helpful tips, reviewing an example, and aiming for a high score, students can excel in these challenging assessments.

A DBQ is an essay-style question that requires students to analyze and interpret historical documents to support their argument. To answer a DBQ effectively:

1. Carefully read and understand the prompt. 2. Analyze each document individually. 3. Identify patterns or themes across the documents. 4. Organize your thoughts into a clear thesis statement. 5. Support your argument with evidence from the documents.

Here are some additional tips to help you tackle DBQs successfully:

1. Familiarize yourself with different types of primary sources like maps, letters, speeches etc., as they may appear in DBQ prompts. 2. Use annotations or highlighters to mark important details within each document. 3. Create an outline before writing your essay to ensure logical flow and organization. 4. Provide context for the time period being discussed using relevant background information.

To illustrate these concepts further, let’s examine an example of a DBQ question: “Evaluate the impact of World War II on women’s roles in society.”

By following the steps outlined earlier—careful reading of documents; identifying patterns; organizing thoughts; supporting arguments—a student might argue that World War II had significant effects on women’s roles due to increased employment opportunities outside traditional domestic spheres.

Scoring well on DBQs requires demonstrating both content knowledge and critical thinking skills. Remember these key points:

1. Accurately interpret primary sources by considering their purpose, audience bias etc., while analyzing them critically 2. Support arguments with specific evidence from multiple documents 3. Address counterarguments or alternative viewpoints if necessary 4. Compose clear essays with coherent structure that include strong introductions, well-developed body paragraphs and concise conclusions.

Mastering  the art of answering document-based questions is essential for success in history and social science exams. By following the tips outlined in this post, you’ll be able to effectively tackle any DBQ that comes your way!

You May Also Like!

Inside GWU George Washington University Admissions

Inside GWU: George Washington University Admissions

Table of Contents1 Introduction to GWU2 Admission Requirements3 Application Process4...

Inside Brown University Demystifying Admission Requirements

Inside Brown University: Demystifying Admission Requirements

Table of Contents1 Introduction to Brown University2 Admission Requirements for...

Emerson College Admission Requirements A Closer Look

Emerson College Admission Requirements: A Closer Look

Table of Contents1 Introduction to Emerson College2 Admission Requirements for...

Bowdoin Admission Requirements Your Path to Excellence

Bowdoin Admission Requirements: Your Path to Excellence

Table of Contents1 Introduction to Bowdoin College2 Admission Process and...

Boston University's Admission Requirements Unraveled

Boston University’s Admission Requirements Unraveled

Table of Contents1 Introduction to Boston University2 Requirements for Undergraduate...

BYU Admission Requirements A Closer Look

BYU Admission Requirements: A Closer Look

Table of Contents1 Introduction to BYU2 Overview of BYU’s Admission...

We Are Here To Help You To Excel in Your Exams!

Book your free demo session now, head office.

  • HD-213, WeWork DLF Forum, Cyber City, DLF Phase 3, DLF, Gurugram, Haryana-122002
  • +919540653900
  • [email protected]

Tychr Online Tutors

IB Online Tutor

Cambridge Online Tutor

Edexcel Online Tutors

AQA Online Tutors

OCR Online Tutors

AP Online Tutors

SAT Online Tuition Classes

ACT Online Tuition Classes

IB Tutor in Bangalore

IB Tutors In Mumbai

IB Tutors In Pune

IB Tutors In Delhi

IB Tutors In Gurgaon

IB Tutors In Noida

IB Tutors In Chennai

Quick Links

Who We Are?

Meet Our Team

Our Results

Our Testimonials

Let’s Connect!

Terms & Conditions

Privacy Policy

Refund Policy

Recent Articles

International ib tutors.

IB Tutor in Singapore

IB Tutor in Toronto

IB Tutor in Seattle

IB Tutor in San Diego

IB Tutor in Vancouver

IB Tutor in London

IB Tutor in Zurich

IB Tutor in Basel

IB Tutor in Lausanne

IB Tutor in Geneva

IB Tutor in Ontario

IB Tutor in Boston

IB Tutor in Kowloon

IB Tutor in Hong Kong

IB Tutor in San Francisco

IB Tutor in Dallas

IB Tutor in Houston

IB Tutor in Chicago

IB Tutor in New York City

IB Tutor in Brooklyn

IB Tutor in Washington

IB Tutor in Berkshire

IB Tutor in Sussex

IB Tutor in Melbourne

IB Tutor in Western Australia

Ⓒ 2023 TYCHR ACADEMY | All Rights Reserved

Announcement.

Download our Successful College Application Guide developed by counselors from the University of Cambridge for institutions like Oxbridge  alongside other Ivy Leagues . To join our college counseling program, call at +918825012255

We are hiring a Business Development Associate and Content Writer and Social Media Strategist at our organisation TYCHR to take over the responsibility of conducting workshops and excelling in new sales territory. View More

thesis statement dbq example

  • AP Calculus
  • AP Chemistry
  • AP U.S. History
  • AP World History
  • Free AP Practice Questions
  • AP Exam Prep

AP World History: Sample DBQ Document Organization

Using the following documents, analyze how the Ottoman government viewed ethnic and religious groups within its empire for the period 1876–1908. Identify an additional document and explain how it would help you analyze the views of the Ottoman Empire.

Analyzing the Documents

The readers award credit based on what the essays accomplish. They do not remove points if an essay is off-task, written poorly, or wrong. There is one exception, however. In the DBQ, you must demonstrate that you understand the documents being used. If your essay makes more than one major misinterpretation, credit cannot be earned.

A major misinterpretation is one that misses the basic intent of the document. If you wrote that the Proclamation of the Young Turks (document 7) was a movement away from ethnic and religious equality, that misinterpretation would be a major error.

If, instead, you wrote that the Ottoman Empire survived for decades after the Proclamation of the Young Turks in 1908, the statement would be wrong (the Ottoman Empire collapsed after World War I) but would not be a misinterpretation of the document. All of the documents could still count as being understood properly. Be careful—especially with visual and graphic documents. Students tend to misinterpret these non-written documents more than they misinterpret traditional written documents.

Kaplan Pro Tip For charts and graphs, pay particular attention to the title and to the factors delineating the information in the visual. This will help you interpret the document. For pictures, remember that all pictures are taken for a reason and reflect the point of view of the photographer and/or the subject. Notice details in the background or foreground that can help you interpret them.

Using Evidence to Support Your Thesis

  • Refer to the document number directly in the sentence: “As shown by document 7, the Young Turks believed that all ethnic and religious groups should be treated equally.”
  • Refer to the document within parentheses at the end of the sentence: “The Young Turks believed that all ethnic and religious groups should be treated equally (document 7).”
  • Refer to information presented in the line of source attribution: “As shown by The Proclamation of the Young Turks in 1908, the Young Turks believed that all ethnic and religious groups should be treated equally.”
  • Combine the last two techniques: “As shown by The Proclamation of the Young Turks in 1908, the Young Turks believed that all ethnic and religious groups should be treated equally (document 7).” (best option)
  • Give no attribution: “The Young Turks believed that all ethnic and religious groups should be treated equally.” (worst option)

Discussing Point of View

  • Does the occupation of the author give the document more or less reliability? For example, government officials may overstate or exaggerate information for political, state, or personal reasons.
  • Does the social class, religion, national background, or gender of the author influence what is mentioned in the document?
  • Does the type of document influence the content of what is said? A journal entry or private letter might be more candid about a topic than a public address that is meant to be persuasive. A political cartoon by definition is exaggerated and meant to convey a certain message, whereas a photograph may accurately represent what was in front of the camera for a shot, but could be staged and framed to capture only a certain perspective.
  • Does the timing of the document influence the message? Recollections and memoirs written long after an event may not have the same reliability as first-hand materials done immediately afterward.
  • Does the intended audience skew the message of a source? If a document is meant to be read by the sultan, it has a different POV than one written for a European audience.
  • Describing the tone of the document can also count for POV if the document is sarcastic, triumphant, haughty, etc. Using tone for POV can be more subtle and is best used with other descriptions of POV.

Grouping Documents Together in Your Analysis

Organizing your documents, using additional historical evidence.

  • a document from the sultan himself since he represents the central power of the Ottoman Empire
  • official orders from the Ottoman government on how to treat different ethnic and religious subjects since such a document could show how the government implemented its policies
  • a chart showing statistics of religious diversity within the empire that would help describe the position of the official faith within the empire
  • a speech or an article from a Young Turk on his attitude toward the Ottoman government that would help show the differences in thought between reformers and officials
  • a document from a religious leader within the Ottoman Empire that would provide a sense of how official religious policies were perceived by the religious communities themselves
  • a map showing the distribution of different ethnic groups within the Ottoman Empire, which would help illustrate the divisions faced by this multi-ethnic country

You might also like

ap-world-history-modern-notes-period-4-apwhm-apwh

Call 1-800-KAP-TEST or email [email protected]

Prep for an Exam

MCAT Test Prep

LSAT Test Prep

GRE Test Prep

GMAT Test Prep

SAT Test Prep

ACT Test Prep

DAT Test Prep

NCLEX Test Prep

USMLE Test Prep

Courses by Location

NCLEX Locations

GRE Locations

SAT Locations

LSAT Locations

MCAT Locations

GMAT Locations

Useful Links

Kaplan Test Prep Contact Us Partner Solutions Work for Kaplan Terms and Conditions Privacy Policy CA Privacy Policy Trademark Directory

If you're seeing this message, it means we're having trouble loading external resources on our website.

If you're behind a web filter, please make sure that the domains *.kastatic.org and *.kasandbox.org are unblocked.

To log in and use all the features of Khan Academy, please enable JavaScript in your browser.

AP®︎/College US History

Course: ap®︎/college us history   >   unit 10.

  • AP US History periods and themes
  • AP US History multiple choice example 1
  • AP US History multiple choice example 2
  • AP US History short answer example 1
  • AP US History short answer example 2
  • AP US History DBQ example 1
  • AP US History DBQ example 2

AP US History DBQ example 3

  • AP US History DBQ example 4
  • AP US History long essay example 1
  • AP US History long essay example 2
  • AP US History long essay example 3
  • Preparing for the AP US History Exam (5/4/2016)
  • AP US History Exam Prep Session (5/1/2017)

thesis statement dbq example

Want to join the conversation?

  • Upvote Button navigates to signup page
  • Downvote Button navigates to signup page
  • Flag Button navigates to signup page

Good Answer

Video transcript

thesis statement dbq example

How to Write a DBQ

thesis statement dbq example

A DBQ essay is an assigned task which tests a student’s analyzation and understanding skills. They also test a student in thinking outside the box. These skills are essential for success in gaining this academic qualification. In this article from EssayPro — professional essay writers team, we will talk about how to write a DBQ, we will go through the DBQ format, and show you a DBQ example.

What Is a DBQ?

Many students may prosper: “What is a DBQ?”. Long story short, DBQ Essay or “Document Based Question” is an assigned academic paper which is part of the AP U.S. History exam (APUSH) set by the United States College Board. It requires a student’s knowledge of a certain topic with evidence from around 3 to 16 reliable sources. Understanding the APUSH DBQ and its outline is essential for success in the exam, itself.

DBQ Outline

We understand that learning how to write a DBQ essay can be difficult for beginners. This is why our professional writers have listed the DBQ format for your own reference while preparing for the exam. Like all essays, this involves an introduction, thesis, body, and conclusion.

How to Write a DBQ

Introduction

  • An introductory sentence to hook your audience.
  • State the background of the topic. Using a source relating to a historical occurrence or historical figure can be helpful at this time.
  • Describe the claims made in your paper which can be supported by the evidence.
  • Create a brief description of the evidence that will be included in the body paragraphs.
  • Write a paragraph which talks about how the DBQ essay question will be answered.

Body Paragraph 1

  • Include the strongest argument. This should be linked to the thesis statement. Read our example of thesis statement .
  • Include an analysis of the references which relate to the strongest argument.
  • Write a statement which concludes the analysis in a different point of view. Include a link to the thesis.
  • Write a transition sentence to the next body paragraph.

Body Paragraph 2

  • Include a reasonable argument which links to the thesis, and the first argument in the previous body paragraph.

Body Paragraph 3

  • Include a reasonable argument which links to the thesis, and the second argument in the previous body paragraph.
  • Write a transition sentence to the conclusion.
  • Create a summarizing argument of the whole paper.
  • Include the main points or important information in the sources.
  • Create a concluding sentence or question which challenges the point of view that argues against these sources.

Feeling Overwhelmed Writing a DBQ ESSAY?

Asking yourself 'who can write my research paper '? Our experts are able to produce an essay within hours.

How to Write a DBQ: Step-By-Step Instructions

For some students, writing a DBQ essay may be hard. Not to worry. Our easy-to-read step-by-step instructions talk about the essential points which includes how to write a DBQ thesis, analyzation, time-management and proofreading your work. It is always important to write your paper in accordance to the DBQ outline for achieving the success you’re capable of.

The DBQ involves:

  • Planning: 15 Minutes
  • Writing: 2 hours and 45 Minutes
  • Proofreading: 10 Minutes

Time management is essential for a successful grade in this form of examination. The general DBQ outline states that the duration is 3 hours and 15 minutes. Spend around 15 minutes planning, 2 hours and 45 minutes writing, and 10 minutes proofreading. Follow these easy-to-read step-by-step instructions to learn how to write a DBQ thesis, body and conclusion successfully.

Step 1: Planning (15 Minutes)

During the exam, it is important to study the provided sources. The exam is 3 hours, so 15 minutes for planning is a reasonable approach. During this time, analyze all of the important key-points from the sources provided. Then, take a note of all of the key points, and write them under the titles; introduction, thesis, body, and conclusion.

Step 2: Introduction (5 Minutes)

First impressions count. Keep the introduction short and brief. Don’t go straight into answering the question in this part of the paper. For a successful introduction, write a brief summary of the overall paper. It is also important to include an introductory sentence.

Step 3: Thesis (20 Minutes)

This form of essay requires a separate 3 paragraphs for the DBQ thesis. Describe the claims made in your paper which can be supported by the evidence. The second paragraph should include a description of the paper. The third paragraph should include how you’re going to answer the question.

  • The key difference with other essays is that the thesis plays an important role in the DBQ structure.
  • The APUSH DBQ thesis should not be two sentences long.
  • The thesis should be written with act least 2 or 3 paragraphs long.

Step 4: Body (2 Hours and 16 Minutes)

Write well-structured, categorized paragraphs. Each paragraph should include one point. Avoid mixing ideas in the paragraphs. Include your answer to the assigned question with the provided documents. It is also important to read between the lines. Each paragraph should link to the thesis.

Step 5: Conclusion (10 Minutes)

The final part of your paper. The conclusion plays a vital role in persuading your audience. A poorly written conclusion means a skeptical audience. For well-written conclusion, summarize the entire paper. Link the conclusion to the thesis. Answer the question in a concluding sentence, “the big idea”.

Step 6: Proofreading (10 Minutes)

Spend around 10 minutes proofreading your work at the end of the exam. It is important to proofread your work to make sure it does not contain any grammatical mistakes. Any writing errors can lower one’s grade. Please make sure that the body paragraphs answer the question and link to the thesis, this is the most important part of the paper.

Writing Tips to Success with Your DBQ Essay

Understand: Before writing, make sure that you understand the sources and the essay question. Duration: Remember that the exam duration is 3 hours and 15 minutes. Study: Practice how to write a DBQ before the actual exam. Identify: Find the key-points from the sources to include in your essay.

How to Write a DBQ

Read Between the Lines: Don’t just write about what you read, but write about what the passages imply. Read all Documents: Make sure you have read all of the sources, prior to writing the paper. Read the Outline: Following the DBQ essay outline is essential for understanding how to structure the paper during the exam. Categorize: Put each point into categories. This will come in useful for writing the body paragraphs. Write the Author’s Opinion: Show an understanding of the writer’s point of view. Write a Temporary DBQ Thesis on your Notes: Doing so will assist you during the paper writing. Follow DBQ Examples: Following a DBQ essay example, while studying, is an excellent way to get a feel for this form of assignment.

DBQ Example

Do you need more help? Following a sample DBQ essay can be very useful for preparation. Usually, when practicing for exams, students commonly refer to an example for understanding the DBQ structure, and other revision purposes. Click on the button to open our DBQ example from one of our professional writers. Feel free to use it as a reference when learning how to write a DBQ.

The Great War and the second ordeal of conflict in Europe, played a fundamental in the increase of the rights for women. During the second world war, the British government encouraged house-wives to do the work of what was primarily traditional for men to do.Such as growing crops and butchering animals, which was generally considered to be“men’s work”. One of the slogans was “dig for victory”. The reason for this was for people to take care of themselves during the difficult times of rationing.

If you think that it's better to pay someone to write my dissertation instead of writing it by your own, get help from our law essay writing team.

Following steps and outlines for custom writing is a great way to learn how to write a DBQ essay. As well as writing tips. Time management is vital for the positive result. Following our advice will enable you to get a good grade by learning how to write a good DBQ. Because learning the DBQ format is essential. Practice is very important for any form of examination. Otherwise, one could not do as well as his or her potential allows him or her to do so.

You might be interested in information about this type of essay, such as the definition essay .

Are you still stuck? Do you sometimes think to yourself: 'Can someone write essay for me '? You’re in luck. Our essay writing service is designed to allow you to easily find custom essay writers at your convenience. Every DBQ essay we deliver is completely original.

Do You Need Help From A PROFESSIONAL ESSAY WRITER?

Our experts are able to produce a DBQ essay example within hours. Why not give it a try to improve your knowledge?

Adam Jason

is an expert in nursing and healthcare, with a strong background in history, law, and literature. Holding advanced degrees in nursing and public health, his analytical approach and comprehensive knowledge help students navigate complex topics. On EssayPro blog, Adam provides insightful articles on everything from historical analysis to the intricacies of healthcare policies. In his downtime, he enjoys historical documentaries and volunteering at local clinics.

thesis statement dbq example

IMAGES

  1. How To Write A Good Dbq Ap World

    thesis statement dbq example

  2. Dbq Thesis Template

    thesis statement dbq example

  3. Dbq Thesis Template

    thesis statement dbq example

  4. how to create a dbq thesis

    thesis statement dbq example

  5. Structure For A DBQ Thesis Format Essay

    thesis statement dbq example

  6. 🌈 Dbq thesis statement generator. FREE Thesis Statement Generator Tool

    thesis statement dbq example

VIDEO

  1. Class 11 Physics

  2. Easy THESIS Trick (DBQ/LEQ)

  3. How To Write A Thesis Statement

  4. How to thesis (w/ Mongols DBQ)

  5. What is thesis statement and example?

  6. IB English: Paper 2

COMMENTS

  1. AP World History: Sample DBQ Thesis Statements

    Let's take a look at a sample AP World History DBQ question and techniques to construct a solid thesis. Using the following documents, analyze how the Ottoman government viewed ethnic and religious groups within its empire for the period 1876-1908. Identify an additional document and explain how it would help you analyze the views of the ...

  2. DBQ Thesis Formula (With AP World & APUSH Thesis Examples!)

    If you're not sure how to write a DBQ thesis, check out this post for a failproof DBQ thesis formula and AP World History and APUSH DBQ thesis examples!

  3. PDF 2022 AP Student Samples and Commentary

    Question 1: Document-Based Question, Imperialism and Asian and African economies 7 points . General Scoring Notes • Except where otherwise noted, each point of these rubrics is earned independently; for example, a student could earn a point for evidence without earning a point for thesis/claim. • Accuracy:

  4. PDF 2019 APUSH DBQ Sample Responses

    SAMPLE RESPONSE E Minimalist SCORE: This 3 essay is included just to demonstrate that a student can score three points (above average) very easily by writing a thesis statement, referencing three documents, and including outside evidence. DOCUMENTS IN BRIEF: DOC 1 Jane Addams describes the political machines of the 1890s in a book published in ...

  5. How to Ace the AP World History DBQ: Rubric, Examples, and Tips

    Step 2: Practice Creating a Thesis. A thesis statement is a sentence or two, located in your essay's introduction, that explains what your essay will be about. ... The second AP World History DBQ example thesis addresses something more complex: how ethnic tensions led to economic exploitation.

  6. How to Write a DBQ Essay: Key Strategies and Tips

    You could even try writing multiple thesis statements in response to the same prompt! It is a great exercise to see how you could approach the prompt from different angles. Time yourself for 5-10 minutes to mimic the time pressure of the AP exam. If possible, have a trusted advisor or friend look over your practice statements and give you feedback.

  7. The Ultimate APUSH DBQ Guide: Rubric, Examples, and More!

    Because your DBQ response will have to choose a position and defend it, you'll need to work on writing strong thesis statements. A thesis statement is essentially your argument in a nutshell, and it sums up the purpose of your essay. The most important aspect of your APUSH DBQ thesis is that it has to make a claim that is both arguable ...

  8. How to Write the Document Based Question (DBQ)

    A Document Based Question (DBQ) is a measure of the skills you learned in your AP classes in regard to recalling history and analyzing related documents. ... Formulate a tentative thesis A thesis is a statement that should be proved and discussed upon. It's important to have a strong thesis as the foundation of your DBQ, as it guides the rest ...

  9. AP World History DBQ Outline + Thesis Practice & Answers

    Eric Beckman. E. Evan Liddle. DBQ Practice is very important when preparing for the AP World exam. It is recommended to write a short brief outline of your argument before writing your body paragraph. Your task: In 20 minutes or less, read the documents and: Outline arguments you would make, using LESS than a full sentence for each.

  10. PDF AP® UNITED STATES HISTORY

    Question 1 — Document-Based Question Explain the reasons why a new conservatism rose to prominence in the United States between 1960 and 1989. A. Thesis: 0-1 point Skills assessed: Argumentation + Causation States a thesis that directly addresses all parts of the question. The thesis must do more than restate the question. 1 point

  11. How to Write a DBQ Essay for APUSH

    The Document Based Question (DBQ) essay is a key feature of the APUSH exam. And at 25% of your total score, it's an important feature! ... Let's walk through a sample DBQ topic for the APUSH exam. ... First paragraph: introduction with a thesis statement; Second paragraph: documents FOR expansion (As you write, make sure to mention who is ...

  12. How to WRITE a THESIS for a DBQ & LEQ [AP World, APUSH, AP Euro]

    Resources from Heimler's History: To master all the WRITING SKILLS you need, get my ESSAY CRAM COURSE: +AP Essay CRAM Course (DBQ, LEQ, SAQ Help): https://bi...

  13. How to Write a DBQ

    This page details all aspects of writing a DBQ including how to earn the contextualization, thesis, evidence, analysis, and sourcing points, how to write a compare & contrast essay, cause & effect essay, and change & continuity over time (CCOT) essay. It also has a free downloadable worksheet linked to it to help you organize your DBQ.

  14. AP US History DBQ example 1 (video)

    AP US History DBQ example 1. Google Classroom. Microsoft Teams. About. Transcript. The document-based question (DBQ) is one of two main essays on the AP US History exam and usually requires analyzing changes or continuities over time in US history. In this video, learn about the structure of DBQs and tips and tricks to help you succeed on this ...

  15. Where to Find the Best DBQ Examples

    One of the best ways to prepare for the DBQ (the "document-based question" on the AP European History, AP US History, and AP World History exams) is to look over sample questions and example essays. Doing this will help you to get a sense of what makes a good (and what makes a bad) DBQ response. That said, not all DBQ essay examples are created equal.

  16. AP US History DBQ example 4 (video)

    For the DBQ we're gonna do a standard five-paragraph essay, which means introduction. . . with a thesis statement, paragraph 1, paragraph 2, paragraph 3. These will all be the body paragraphs where we'll give our evidence in support of our thesis. And then a conclusion, which wraps up what we've said before, and really drives home our point.

  17. APUSH Document Based Questions and Responses: A Study Guide

    Not at all. There could be thousands of things that go under economics, politics, and society - and many things could be considered "unfair" - so I have no idea what the student will be arguing in this DBQ. Thesis Statement Dos and Don'ts. DO: Directly answer the question being asked. Be specific. Write a thesis statement like the ...

  18. How to Write a DBQ Essay (with Pictures)

    2. Identify the prompt's keywords and assigned tasks. Ensure you understand what evidence to look for in the documents and what your essay needs to accomplish. Circle or underline task-oriented words such as "evaluate," "analyze," and "compare and contrast.".

  19. AP US History DBQ example 2 (video)

    - [Voiceover] In this video we're talking some more about the DBQ or document-based question section of the AP U.S. history exam. In our first video we just went through some general strategy about how to approach the question, which asks you to write an essay with a thesis statement addressing this question about the social and economic experiences of African Americans moving from the South ...

  20. DBQ Example: How to Tackle Document-Based Questions

    Analyze each document individually. 3. Identify patterns or themes across the documents. 4. Organize your thoughts into a clear thesis statement. 5. Support your argument with evidence from the documents. Here are some additional tips to help you tackle DBQs successfully: 1.

  21. AP World History: Sample DBQ Document Organization

    Now that you've crafted a thesis statement, you'll need to develop a plan for how to incorporate the given documents into your DBQ essay. Review the prompt and the documents below: ... Be careful: mentioning a type of document that you have already been given disqualifies the statement. For our sample DBQ, mentioning a document from a ...

  22. AP US History DBQ example 3 (video)

    AP US History DBQ example 1. AP US History DBQ example 2. AP US History DBQ example 3. AP US History DBQ example 4. AP US History long essay example 1. ... So I think if there is a thesis statement to be gotten out of this, it's that moving from the South to the North, African-Americans in the period from 1910 to 1930, experience pretty much ...

  23. How to Write a DBQ: Definition, Step-By-Step, & DBQ Example

    Step 3: Thesis (20 Minutes) This form of essay requires a separate 3 paragraphs for the DBQ thesis. Describe the claims made in your paper which can be supported by the evidence. The second paragraph should include a description of the paper. The third paragraph should include how you're going to answer the question.